Cómo contestar las preguntas excepto argumento en la lsat

Muchos fortalecer / debilitar preguntas sobre el LSAT no piden la opción que mejor se fortalece o debilita (o soportes, o socava) el argumento pero en vez de la elección que no hace hacer una de esas cosas. Estas preguntas terminan en la palabra de suma importancia EXCEPTO. Siempre ha capitalizado, por lo que no se puede perder.

Estas preguntas tienen el siguiente aspecto:

  • Cada uno de los siguientes, si es cierto, apoya el argumento anterior, excepto:

  • Cada uno de los siguientes, si es cierto, debilitaría el argumento de que el comentarista SALVO:

  • Cada uno de los siguientes, si es cierto, apoya la hipótesis de que el físico SALVO:

  • Cada uno de los siguientes, si es cierto, debilita el argumento SALVO:

  • Cada uno de los siguientes, si es cierto, refuerza el argumento SALVO:

Video: Argumentación y Retórica - Clase 7

También pueden tener este aspecto:

  • ¿Cuál de los siguientes, si todos ellos son verdaderas, el más pequeño de utilidad para establecer que la anterior conclusión se extrae correctamente?

  • ¿Cuál de los siguientes, si todos ellos son verdaderas, es menos eficaces a socavar el argumento de los políticos?

Video: PREGUNTAS Y RESPUESTAS Y RETOS ESTUPIDOS #3

Todavía están fortalecen / debilitan preguntas, pero en lugar de encontrar una respuesta para fortalecer / debilitar / soporte / minar, que desea encontrar cuatro.

Como siempre, leer primero la pregunta cuidadosamente. Estas preguntas no son el estándar de fortalecer / debilitar preguntas. En su lugar, ofrecen cuatro opciones que fortalecen o debilitan la conclusión, y uno que no lo hace. Usted no necesariamente tiene que encontrar una respuesta que fortalece o debilita la conclusión: lo que desea es una respuesta que no hace lo que los otros cuatro lo hacen.

Video: PREGUNTAS INCÓMODAS QUE MOTIVARÁN A MUCHOS TESTIGOS DE JEHOVÁ A CUESTIONAR SU FE

Por ejemplo, si la pregunta dice: “Cada uno de los siguientes, si es cierto, debilita el argumento SALVO:” la respuesta correcta no necesariamente a reforzar el argumento, pero definitivamente no debilitarlo.

Aquí está un ejemplo de una pregunta que tiene cuatro respuestas que debilitan el argumento y uno que no lo hace:

Algunos trabajadores de las fábricas de palomitas de maíz de microondas han contraído una enfermedad pulmonar rara. Los expertos han vinculado esta enfermedad a un producto químico utilizado en el proceso de mezcla de palomitas de maíz y saborizantes. Los consumidores deben, por lo tanto, dejar de comprar y comer todo tipo de palomitas de microondas para evitar el riesgo de contraer esta enfermedad pulmonar.

Cada uno de los siguientes, si es cierto, debilita el argumento SALVO:

  • (A) La enfermedad pulmonar es causado por la exposición a la sustancia química durante muchas horas a la vez durante un período de años.

  • (B) El producto químico sólo se convierte en tóxico cuando se mantiene a una temperatura mucho mayor que la alcanzada por las palomitas de maíz en un microondas.

  • (C) La enfermedad pulmonar sólo se ha encontrado en los trabajadores que manejan el sabor de especias cajún de palomitas de maíz.

  • (D) En 20 años de consumo generalizado palomitas de microondas, ningún consumidor ha contratado alguna vez esta enfermedad pulmonar rara.

    Video: American Ninja Warrior Vs. Project Runway • Pirate Stunt Show

  • (E) La EPA todavía no ha hecho ninguna investigación para determinar si el producto químico que causa la enfermedad pulmonar está presente en el vapor y el aire que sale de una bolsa de palomitas de maíz cuando se abre.

De la lectura de la pregunta en primer lugar, usted sabe que quiere una respuesta que no debilita el argumento. La conclusión es que los consumidores deben dejar de comer todo tipo de palomitas de microondas para evitar contraer una enfermedad. La evidencia de esta conclusión es que un producto químico en las palomitas de microondas causó esta enfermedad en los empleados de la fábrica de palomitas de maíz.

Lo que debilita el argumento? Cualquier evidencia de que hace que los consumidores parecen estar a salvo. Si se puede demostrar que la enfermedad es de alguna manera restringida a los trabajadores de las fábricas, se puede asegurar a los consumidores que pueden comer las palomitas de microondas con impunidad. Lo que no debilita la conclusión? Cualquier evidencia que indica que los consumidores pueden contraer la enfermedad por comer palomitas de microondas, o cualquier evidencia de que es de relevancia claro o simplemente fuera de tema.

  • Choice (A) debilita el argumento porque la mayoría de los consumidores no están expuestos a la sustancia química en cuestión durante largos períodos de tiempo.

  • La opción (B) debilita el argumento porque los consumidores no están manteniendo sus palomitas de maíz lo suficientemente caliente.

  • Opción (C) debilita la sugerencia de que los consumidores deben evitar todo tipo de palomitas de microondas, aunque posiblemente no la sugerencia de que deben evitar el sabor de especias cajún.

  • Choice (D) debilita la sugerencia porque hace que el riesgo para los consumidores parecen casi insignificante.

  • Choice (E) no debilita el argumento, si el EPA aún no se ha hecho ninguna investigación en la presencia de la sustancia química en vapor palomitas de maíz, entonces la química bien podría estar al acecho allí, listo para enfermar a los consumidores desafortunados.

Choice (E) no especialmente reforzar el argumento, pero no debilita, tampoco. Choice (E) es la respuesta correcta.

Artículos Relacionados